Finding a 2x2 Matrix raised to the power of 1000












4














Let $A= pmatrix{1&4\ 3&2}$. Find $A^{1000}$.



Does this problem have to do with eigenvalues or is there another formula that is specific to 2x2 matrices?










share|cite|improve this question




















  • 5




    Eigenvalues should do it. Does a formula exist? Probably, but it would just be the process of diagonalizing, written out in one line.
    – Eric Stucky
    Dec 8 '13 at 2:17












  • Unless there is a pattern specific to the matrix, usually one has to the diagonalize it.
    – David H
    Dec 8 '13 at 2:18










  • Abstract duplicate? of math.stackexchange.com/questions/55285/…
    – Eric Stucky
    Dec 8 '13 at 2:23


















4














Let $A= pmatrix{1&4\ 3&2}$. Find $A^{1000}$.



Does this problem have to do with eigenvalues or is there another formula that is specific to 2x2 matrices?










share|cite|improve this question




















  • 5




    Eigenvalues should do it. Does a formula exist? Probably, but it would just be the process of diagonalizing, written out in one line.
    – Eric Stucky
    Dec 8 '13 at 2:17












  • Unless there is a pattern specific to the matrix, usually one has to the diagonalize it.
    – David H
    Dec 8 '13 at 2:18










  • Abstract duplicate? of math.stackexchange.com/questions/55285/…
    – Eric Stucky
    Dec 8 '13 at 2:23
















4












4








4


5





Let $A= pmatrix{1&4\ 3&2}$. Find $A^{1000}$.



Does this problem have to do with eigenvalues or is there another formula that is specific to 2x2 matrices?










share|cite|improve this question















Let $A= pmatrix{1&4\ 3&2}$. Find $A^{1000}$.



Does this problem have to do with eigenvalues or is there another formula that is specific to 2x2 matrices?







linear-algebra matrices diagonalization






share|cite|improve this question















share|cite|improve this question













share|cite|improve this question




share|cite|improve this question








edited Dec 8 '13 at 2:23









Julien

38.5k358129




38.5k358129










asked Dec 8 '13 at 2:16









user114220

59117




59117








  • 5




    Eigenvalues should do it. Does a formula exist? Probably, but it would just be the process of diagonalizing, written out in one line.
    – Eric Stucky
    Dec 8 '13 at 2:17












  • Unless there is a pattern specific to the matrix, usually one has to the diagonalize it.
    – David H
    Dec 8 '13 at 2:18










  • Abstract duplicate? of math.stackexchange.com/questions/55285/…
    – Eric Stucky
    Dec 8 '13 at 2:23
















  • 5




    Eigenvalues should do it. Does a formula exist? Probably, but it would just be the process of diagonalizing, written out in one line.
    – Eric Stucky
    Dec 8 '13 at 2:17












  • Unless there is a pattern specific to the matrix, usually one has to the diagonalize it.
    – David H
    Dec 8 '13 at 2:18










  • Abstract duplicate? of math.stackexchange.com/questions/55285/…
    – Eric Stucky
    Dec 8 '13 at 2:23










5




5




Eigenvalues should do it. Does a formula exist? Probably, but it would just be the process of diagonalizing, written out in one line.
– Eric Stucky
Dec 8 '13 at 2:17






Eigenvalues should do it. Does a formula exist? Probably, but it would just be the process of diagonalizing, written out in one line.
– Eric Stucky
Dec 8 '13 at 2:17














Unless there is a pattern specific to the matrix, usually one has to the diagonalize it.
– David H
Dec 8 '13 at 2:18




Unless there is a pattern specific to the matrix, usually one has to the diagonalize it.
– David H
Dec 8 '13 at 2:18












Abstract duplicate? of math.stackexchange.com/questions/55285/…
– Eric Stucky
Dec 8 '13 at 2:23






Abstract duplicate? of math.stackexchange.com/questions/55285/…
– Eric Stucky
Dec 8 '13 at 2:23












6 Answers
6






active

oldest

votes


















4














You should diagonalize it, if possible.



This means to find the eigenvalues and eigenvectors.




  1. The eigenvalues are roots of the characteristic polynomial of $A$, which is
    $$detpmatrix{1-x&4\3&2-x}=(1-x)(2-x)-12=x^2-3x-10,.$$

  2. If you find its roots, (which together sum up to $3$ and multiply to $-10$), then find an eigenvector for both ($v_1$ and $v_2$), [e.g. $v_2:=pmatrix{1\1}$ will be an eigenvector].

  3. Then build the matrix $P$ with columns $(v_1|v_2)$, and calculate its inverse.

  4. Finally, $D:=P^{-1}AP$ will be the diagonal containing the eigenvalues, because $$AP=A,(v_1|v_2)=(lambda_1 v_1,|,lambda_2 v_2) = P,pmatrix{lambda_1&0\
    0&lambda_2}$$


And after all these, you can easily raise $A$ to any power:
$$A^{1000}=PD^{1000}P^{-1},.$$






share|cite|improve this answer





























    7














    Check that you get $A=PBP^{-1}$ with $B$ a diagonal matrix with entries $-2$ and $5$, and $P$ an invertible matrix. Then note that $A^2=(PBP^{-1})(PBP^{-1})=PB^2P^{-1}$, and in general you get $$A^n=PB^nP^{-1}$$



    The right hand side is easy to deal with since the power of a diagonal matrix is very easy to see :)






    share|cite|improve this answer





























      4














      The well-known strategy is to diagonalize since this matrix is diagonalizable.
      For a change, here is a slightly different approach which is convenient for small matrices. If you have time, try to perform both methods until the end without computer help. This is fairly equivalent. Although you have two linear systems to solve for eigenvectors, and only one here for the constants $c,d$ below.



      We will compute $A^n$.



      Let $p_A(X)=X^2-3X-10$ be the characteristic polynomial of the matrix $A$. By Cayley-Hamilton, $p_A(A)=0$. So it suffices to determine the degree $leq 1$ remainder in the Euclidean divison of $X^{n}$ by $p_A(X)$ to compute $A^{n}$.



      For every $ngeq 1$, denote




      $$
      X^n=q_n(X)p_A(X)+a_nX+b_n
      $$




      the Euclidean division of $X^n$ by $p_A(X)$.
      Since $X^2equiv 3X+10$ modulo $p_A(X)$, multiplication of the latter by $X$ yields
      $$
      cases{a_{n=1}=3a_n+b_n \b_{n+1}=10a_n}iffcases{a_{n+1}=3a_n+10a_{n-1}\b_{n+1}=10a_n}
      $$
      Solving for $a_n$ is straightforward given the theory of recurrent linear homogeneous sequences. Given that the roots of the characteristic equation (which is of course $p_A(X)=0$) are $-2$ and $5$, we have $a_n=c (-2)^n+d 5^n$. Considering the initial cases $n=0,1$, we find $c$ and $d$ whence




      $$a_n=frac{5^n-(-2)^{n}}{7}quadmbox{whence} quad b_n=frac{2cdot 5^{n}+5(-2)^{n}}{7}$$




      Therefore
      $$
      A^n=q_n(A)p_A(A)+a_nA+b_nI_2=a_nA+b_nI_2=pmatrix{a_n+b_n&4a_n\3a_n& 2a_n+b_n}
      $$
      hence $A^n$ is equal to




      $$A^n=pmatrix{frac{3cdot 5^{n}+ (-2)^{n+2}}{7} & frac{4cdot 5^{n}- (-2)^{n+2}}{7} \ frac{3cdot 5^{n}-3cdot (-2)^{n}}{7} & frac{4cdot 5^{n}+3cdot (-2)^{n}}{7}}$$







      share|cite|improve this answer































        3














        Perform an eigenvalue decomposition of $A$, we then get
        $$A =
        begin{bmatrix}
        -4/5 & -1/sqrt2\
        3/5 & -1/sqrt2
        end{bmatrix}
        begin{bmatrix}
        -2 & 0\
        0 & 5
        end{bmatrix}
        begin{bmatrix}
        -4/5 & -1/sqrt2\
        3/5 & -1/sqrt2
        end{bmatrix}^{-1}
        =VDV^{-1}
        $$
        where $V = begin{bmatrix}
        -4/5 & -1/sqrt2\
        3/5 & -1/sqrt2
        end{bmatrix}$ and $D = begin{bmatrix}
        -2 & 0\
        0 & 5
        end{bmatrix}$.



        Hence,
        $$A^n = underbrace{left(VDV^{-1} right)left(VDV^{-1} right)cdots left(VDV^{-1} right)}_{n text{ times}} = VD^n V^{-1}$$






        share|cite|improve this answer





















        • This is a Jordan decomposition, not an eigenvalue decomposition (nor does an eigenvalue decomposition with a diagonal $D$ exist for non-symmetric/non-Hermitian matrix $A$), because $$V^*V = begin{bmatrix} 1 & (5sqrt{2})^{-1} \ (5sqrt{2})^{-1} & 1 end{bmatrix} ne {rm I}_2.$$
          – Vedran Šego
          Dec 8 '13 at 3:28



















        0














        $newcommand{+}{^{dagger}}%
        newcommand{angles}[1]{leftlangle #1 rightrangle}%
        newcommand{braces}[1]{leftlbrace #1 rightrbrace}%
        newcommand{bracks}[1]{leftlbrack #1 rightrbrack}%
        newcommand{ceil}[1]{,leftlceil #1 rightrceil,}%
        newcommand{dd}{{rm d}}%
        newcommand{ds}[1]{displaystyle{#1}}%
        newcommand{equalby}[1]{{#1 atop {= atop vphantom{huge A}}}}%
        newcommand{expo}[1]{,{rm e}^{#1},}%
        newcommand{fermi}{,{rm f}}%
        newcommand{floor}[1]{,leftlfloor #1 rightrfloor,}%
        newcommand{half}{{1 over 2}}%
        newcommand{ic}{{rm i}}%
        newcommand{iff}{Longleftrightarrow}
        newcommand{imp}{Longrightarrow}%
        newcommand{isdiv}{,left.rightvert,}%
        newcommand{ket}[1]{leftvert #1rightrangle}%
        newcommand{ol}[1]{overline{#1}}%
        newcommand{pars}[1]{left( #1 right)}%
        newcommand{partiald}[3]{frac{partial^{#1} #2}{partial #3^{#1}}}
        newcommand{pp}{{cal P}}%
        newcommand{root}[2]{,sqrt[#1]{,#2,},}%
        newcommand{sech}{,{rm sech}}%
        newcommand{sgn}{,{rm sgn}}%
        newcommand{totald}[3]{frac{{rm d}^{#1} #2}{{rm d} #3^{#1}}}
        newcommand{ul}[1]{underline{#1}}%
        newcommand{verts}[1]{leftvert, #1 ,rightvert}$
        $$
        A = pars{begin{array}{cc}1 & 4 \ 3 & 2end{array}}
        =
        {3 over 2}
        overbrace{pars{begin{array}{cc}1 & 0 \ 0 & 1end{array}}}^{ds{1}}
        +
        overbrace{{7 over 2}}^{ds{b_{x}}}
        overbrace{pars{begin{array}{cc}0 & 1 \ 1 & 0end{array}}}^{ds{sigma_{x}}}
        +
        overbrace{half,ic}^{ds{b_{y}}}
        overbrace{pars{begin{array}{cc}0 & -ic \ ic & 0end{array}}}^{ds{sigma_{y}}}
        overbrace{-
        half}^{ds{b_{z}}}overbrace{pars{begin{array}{cc}1 & 0 \ 1 & -1end{array}}}^{ds{sigma_{z}}}
        =
        {3 over 2} + vec{b}cdotvec{sigma}
        $$
        where $braces{sigma_{ell},, ell = x, y, z}$ are the
        Pauli matrices. Then,
        $expo{At} = expo{3t/2}expo{vec{b}cdotvec{sigma}t}$. However, $vec{b}cdotvec{sigma}vec{b}cdotvec{sigma} = vec{b}cdotvec{b} = 49/4$ such that
        $$
        pars{totald[2]{}{t} - {49 over 4}}expo{vec{b}cdotvec{sigma}t} = 0
        quadimpquad
        expo{vec{b}cdotvec{sigma}t} = muexpo{7t/2} + nuexpo{-7t/2},,quad
        mu, nu mbox{are constants},
        $$
        with $1 = mu + nu$ and $2vec{b}cdotvec{sigma}/7 = mu - nu$ such that
        $mu = 1/2 + vec{b}cdotvec{sigma}/7$ and
        $nu = 1/2 - vec{b}cdotvec{sigma}/7$:
        $$
        expo{At} = muexpo{5t} + nuexpo{-2t},,
        quadsum_{n = 0}^{infty}{t^{n} over n!},A^{n}
        =
        sum_{n = 0}^{infty}{t^{n} over n!},
        bracks{5^{n}mu + pars{-1}^{n}2^{n}nu}
        $$
        $$color{#0000ff}{large%
        leftlbrace%
        begin{array}{rcl}
        A^{n} = 5^{n}mu + pars{-1}^{n}2^{n}nu
        & = &
        halfbracks{5^{n} + pars{-1}^{n}2^{n}}
        +
        {1 over 7}bracks{5^{n} - pars{-1}^{n}2^{n}}vec{b}cdotvec{sigma}
        \[3mm]
        vec{b}cdotvec{sigma}
        & = &
        A - {3 over 2}
        =
        pars{begin{array}{cc}-1/2 & 5/2 \ 3/2 & 1/2end{array}}
        =
        halfpars{begin{array}{cc}-1 & 5 \ 3 & 1end{array}}
        end{array}right.}
        $$






        share|cite|improve this answer































          0














          Two additional methods that you can use once you know the eigenvalues $lambda_1$ and $lambda_2$:




          • Decompose $A$ into $lambda_1P_1+lambda_2P_2$, where $P_1 = {A-lambda_2Ioverlambda_2-lambda_1}$ and $P_2={A-lambda_1Ioverlambda_1-lambda_2}$ are projections onto the corresponding eigenspaces with $P_1P_2=P_2P_1=0$. If you expand $A^{1000}$ using the binomial theorem, you’ll find that all but two terms vanish, giving $lambda_1^{1000}P_1+lambda_2^{1000}P_2$.

          • Use the Cayley-Hamilton theorem to write $A^{1000}=aI+bA$ for some undetermined coefficients $a$ and $b$, then use the fact that this equation is also satisfied by the eigenvalues, which gives you the system of linear equations $a+blambda_i=lambda_i^{100}$ to solve for $a$ and $b$.


          When $A$ has repeated eigenvalues, you’ll need to modify the above methods a bit, but the underlying ideas are still the same.






          share|cite|improve this answer





















            Your Answer





            StackExchange.ifUsing("editor", function () {
            return StackExchange.using("mathjaxEditing", function () {
            StackExchange.MarkdownEditor.creationCallbacks.add(function (editor, postfix) {
            StackExchange.mathjaxEditing.prepareWmdForMathJax(editor, postfix, [["$", "$"], ["\\(","\\)"]]);
            });
            });
            }, "mathjax-editing");

            StackExchange.ready(function() {
            var channelOptions = {
            tags: "".split(" "),
            id: "69"
            };
            initTagRenderer("".split(" "), "".split(" "), channelOptions);

            StackExchange.using("externalEditor", function() {
            // Have to fire editor after snippets, if snippets enabled
            if (StackExchange.settings.snippets.snippetsEnabled) {
            StackExchange.using("snippets", function() {
            createEditor();
            });
            }
            else {
            createEditor();
            }
            });

            function createEditor() {
            StackExchange.prepareEditor({
            heartbeatType: 'answer',
            autoActivateHeartbeat: false,
            convertImagesToLinks: true,
            noModals: true,
            showLowRepImageUploadWarning: true,
            reputationToPostImages: 10,
            bindNavPrevention: true,
            postfix: "",
            imageUploader: {
            brandingHtml: "Powered by u003ca class="icon-imgur-white" href="https://imgur.com/"u003eu003c/au003e",
            contentPolicyHtml: "User contributions licensed under u003ca href="https://creativecommons.org/licenses/by-sa/3.0/"u003ecc by-sa 3.0 with attribution requiredu003c/au003e u003ca href="https://stackoverflow.com/legal/content-policy"u003e(content policy)u003c/au003e",
            allowUrls: true
            },
            noCode: true, onDemand: true,
            discardSelector: ".discard-answer"
            ,immediatelyShowMarkdownHelp:true
            });


            }
            });














            draft saved

            draft discarded


















            StackExchange.ready(
            function () {
            StackExchange.openid.initPostLogin('.new-post-login', 'https%3a%2f%2fmath.stackexchange.com%2fquestions%2f597602%2ffinding-a-2x2-matrix-raised-to-the-power-of-1000%23new-answer', 'question_page');
            }
            );

            Post as a guest















            Required, but never shown

























            6 Answers
            6






            active

            oldest

            votes








            6 Answers
            6






            active

            oldest

            votes









            active

            oldest

            votes






            active

            oldest

            votes









            4














            You should diagonalize it, if possible.



            This means to find the eigenvalues and eigenvectors.




            1. The eigenvalues are roots of the characteristic polynomial of $A$, which is
              $$detpmatrix{1-x&4\3&2-x}=(1-x)(2-x)-12=x^2-3x-10,.$$

            2. If you find its roots, (which together sum up to $3$ and multiply to $-10$), then find an eigenvector for both ($v_1$ and $v_2$), [e.g. $v_2:=pmatrix{1\1}$ will be an eigenvector].

            3. Then build the matrix $P$ with columns $(v_1|v_2)$, and calculate its inverse.

            4. Finally, $D:=P^{-1}AP$ will be the diagonal containing the eigenvalues, because $$AP=A,(v_1|v_2)=(lambda_1 v_1,|,lambda_2 v_2) = P,pmatrix{lambda_1&0\
              0&lambda_2}$$


            And after all these, you can easily raise $A$ to any power:
            $$A^{1000}=PD^{1000}P^{-1},.$$






            share|cite|improve this answer


























              4














              You should diagonalize it, if possible.



              This means to find the eigenvalues and eigenvectors.




              1. The eigenvalues are roots of the characteristic polynomial of $A$, which is
                $$detpmatrix{1-x&4\3&2-x}=(1-x)(2-x)-12=x^2-3x-10,.$$

              2. If you find its roots, (which together sum up to $3$ and multiply to $-10$), then find an eigenvector for both ($v_1$ and $v_2$), [e.g. $v_2:=pmatrix{1\1}$ will be an eigenvector].

              3. Then build the matrix $P$ with columns $(v_1|v_2)$, and calculate its inverse.

              4. Finally, $D:=P^{-1}AP$ will be the diagonal containing the eigenvalues, because $$AP=A,(v_1|v_2)=(lambda_1 v_1,|,lambda_2 v_2) = P,pmatrix{lambda_1&0\
                0&lambda_2}$$


              And after all these, you can easily raise $A$ to any power:
              $$A^{1000}=PD^{1000}P^{-1},.$$






              share|cite|improve this answer
























                4












                4








                4






                You should diagonalize it, if possible.



                This means to find the eigenvalues and eigenvectors.




                1. The eigenvalues are roots of the characteristic polynomial of $A$, which is
                  $$detpmatrix{1-x&4\3&2-x}=(1-x)(2-x)-12=x^2-3x-10,.$$

                2. If you find its roots, (which together sum up to $3$ and multiply to $-10$), then find an eigenvector for both ($v_1$ and $v_2$), [e.g. $v_2:=pmatrix{1\1}$ will be an eigenvector].

                3. Then build the matrix $P$ with columns $(v_1|v_2)$, and calculate its inverse.

                4. Finally, $D:=P^{-1}AP$ will be the diagonal containing the eigenvalues, because $$AP=A,(v_1|v_2)=(lambda_1 v_1,|,lambda_2 v_2) = P,pmatrix{lambda_1&0\
                  0&lambda_2}$$


                And after all these, you can easily raise $A$ to any power:
                $$A^{1000}=PD^{1000}P^{-1},.$$






                share|cite|improve this answer












                You should diagonalize it, if possible.



                This means to find the eigenvalues and eigenvectors.




                1. The eigenvalues are roots of the characteristic polynomial of $A$, which is
                  $$detpmatrix{1-x&4\3&2-x}=(1-x)(2-x)-12=x^2-3x-10,.$$

                2. If you find its roots, (which together sum up to $3$ and multiply to $-10$), then find an eigenvector for both ($v_1$ and $v_2$), [e.g. $v_2:=pmatrix{1\1}$ will be an eigenvector].

                3. Then build the matrix $P$ with columns $(v_1|v_2)$, and calculate its inverse.

                4. Finally, $D:=P^{-1}AP$ will be the diagonal containing the eigenvalues, because $$AP=A,(v_1|v_2)=(lambda_1 v_1,|,lambda_2 v_2) = P,pmatrix{lambda_1&0\
                  0&lambda_2}$$


                And after all these, you can easily raise $A$ to any power:
                $$A^{1000}=PD^{1000}P^{-1},.$$







                share|cite|improve this answer












                share|cite|improve this answer



                share|cite|improve this answer










                answered Dec 8 '13 at 2:35









                Berci

                59.7k23672




                59.7k23672























                    7














                    Check that you get $A=PBP^{-1}$ with $B$ a diagonal matrix with entries $-2$ and $5$, and $P$ an invertible matrix. Then note that $A^2=(PBP^{-1})(PBP^{-1})=PB^2P^{-1}$, and in general you get $$A^n=PB^nP^{-1}$$



                    The right hand side is easy to deal with since the power of a diagonal matrix is very easy to see :)






                    share|cite|improve this answer


























                      7














                      Check that you get $A=PBP^{-1}$ with $B$ a diagonal matrix with entries $-2$ and $5$, and $P$ an invertible matrix. Then note that $A^2=(PBP^{-1})(PBP^{-1})=PB^2P^{-1}$, and in general you get $$A^n=PB^nP^{-1}$$



                      The right hand side is easy to deal with since the power of a diagonal matrix is very easy to see :)






                      share|cite|improve this answer
























                        7












                        7








                        7






                        Check that you get $A=PBP^{-1}$ with $B$ a diagonal matrix with entries $-2$ and $5$, and $P$ an invertible matrix. Then note that $A^2=(PBP^{-1})(PBP^{-1})=PB^2P^{-1}$, and in general you get $$A^n=PB^nP^{-1}$$



                        The right hand side is easy to deal with since the power of a diagonal matrix is very easy to see :)






                        share|cite|improve this answer












                        Check that you get $A=PBP^{-1}$ with $B$ a diagonal matrix with entries $-2$ and $5$, and $P$ an invertible matrix. Then note that $A^2=(PBP^{-1})(PBP^{-1})=PB^2P^{-1}$, and in general you get $$A^n=PB^nP^{-1}$$



                        The right hand side is easy to deal with since the power of a diagonal matrix is very easy to see :)







                        share|cite|improve this answer












                        share|cite|improve this answer



                        share|cite|improve this answer










                        answered Dec 8 '13 at 2:22









                        Daniel Montealegre

                        4,7731443




                        4,7731443























                            4














                            The well-known strategy is to diagonalize since this matrix is diagonalizable.
                            For a change, here is a slightly different approach which is convenient for small matrices. If you have time, try to perform both methods until the end without computer help. This is fairly equivalent. Although you have two linear systems to solve for eigenvectors, and only one here for the constants $c,d$ below.



                            We will compute $A^n$.



                            Let $p_A(X)=X^2-3X-10$ be the characteristic polynomial of the matrix $A$. By Cayley-Hamilton, $p_A(A)=0$. So it suffices to determine the degree $leq 1$ remainder in the Euclidean divison of $X^{n}$ by $p_A(X)$ to compute $A^{n}$.



                            For every $ngeq 1$, denote




                            $$
                            X^n=q_n(X)p_A(X)+a_nX+b_n
                            $$




                            the Euclidean division of $X^n$ by $p_A(X)$.
                            Since $X^2equiv 3X+10$ modulo $p_A(X)$, multiplication of the latter by $X$ yields
                            $$
                            cases{a_{n=1}=3a_n+b_n \b_{n+1}=10a_n}iffcases{a_{n+1}=3a_n+10a_{n-1}\b_{n+1}=10a_n}
                            $$
                            Solving for $a_n$ is straightforward given the theory of recurrent linear homogeneous sequences. Given that the roots of the characteristic equation (which is of course $p_A(X)=0$) are $-2$ and $5$, we have $a_n=c (-2)^n+d 5^n$. Considering the initial cases $n=0,1$, we find $c$ and $d$ whence




                            $$a_n=frac{5^n-(-2)^{n}}{7}quadmbox{whence} quad b_n=frac{2cdot 5^{n}+5(-2)^{n}}{7}$$




                            Therefore
                            $$
                            A^n=q_n(A)p_A(A)+a_nA+b_nI_2=a_nA+b_nI_2=pmatrix{a_n+b_n&4a_n\3a_n& 2a_n+b_n}
                            $$
                            hence $A^n$ is equal to




                            $$A^n=pmatrix{frac{3cdot 5^{n}+ (-2)^{n+2}}{7} & frac{4cdot 5^{n}- (-2)^{n+2}}{7} \ frac{3cdot 5^{n}-3cdot (-2)^{n}}{7} & frac{4cdot 5^{n}+3cdot (-2)^{n}}{7}}$$







                            share|cite|improve this answer




























                              4














                              The well-known strategy is to diagonalize since this matrix is diagonalizable.
                              For a change, here is a slightly different approach which is convenient for small matrices. If you have time, try to perform both methods until the end without computer help. This is fairly equivalent. Although you have two linear systems to solve for eigenvectors, and only one here for the constants $c,d$ below.



                              We will compute $A^n$.



                              Let $p_A(X)=X^2-3X-10$ be the characteristic polynomial of the matrix $A$. By Cayley-Hamilton, $p_A(A)=0$. So it suffices to determine the degree $leq 1$ remainder in the Euclidean divison of $X^{n}$ by $p_A(X)$ to compute $A^{n}$.



                              For every $ngeq 1$, denote




                              $$
                              X^n=q_n(X)p_A(X)+a_nX+b_n
                              $$




                              the Euclidean division of $X^n$ by $p_A(X)$.
                              Since $X^2equiv 3X+10$ modulo $p_A(X)$, multiplication of the latter by $X$ yields
                              $$
                              cases{a_{n=1}=3a_n+b_n \b_{n+1}=10a_n}iffcases{a_{n+1}=3a_n+10a_{n-1}\b_{n+1}=10a_n}
                              $$
                              Solving for $a_n$ is straightforward given the theory of recurrent linear homogeneous sequences. Given that the roots of the characteristic equation (which is of course $p_A(X)=0$) are $-2$ and $5$, we have $a_n=c (-2)^n+d 5^n$. Considering the initial cases $n=0,1$, we find $c$ and $d$ whence




                              $$a_n=frac{5^n-(-2)^{n}}{7}quadmbox{whence} quad b_n=frac{2cdot 5^{n}+5(-2)^{n}}{7}$$




                              Therefore
                              $$
                              A^n=q_n(A)p_A(A)+a_nA+b_nI_2=a_nA+b_nI_2=pmatrix{a_n+b_n&4a_n\3a_n& 2a_n+b_n}
                              $$
                              hence $A^n$ is equal to




                              $$A^n=pmatrix{frac{3cdot 5^{n}+ (-2)^{n+2}}{7} & frac{4cdot 5^{n}- (-2)^{n+2}}{7} \ frac{3cdot 5^{n}-3cdot (-2)^{n}}{7} & frac{4cdot 5^{n}+3cdot (-2)^{n}}{7}}$$







                              share|cite|improve this answer


























                                4












                                4








                                4






                                The well-known strategy is to diagonalize since this matrix is diagonalizable.
                                For a change, here is a slightly different approach which is convenient for small matrices. If you have time, try to perform both methods until the end without computer help. This is fairly equivalent. Although you have two linear systems to solve for eigenvectors, and only one here for the constants $c,d$ below.



                                We will compute $A^n$.



                                Let $p_A(X)=X^2-3X-10$ be the characteristic polynomial of the matrix $A$. By Cayley-Hamilton, $p_A(A)=0$. So it suffices to determine the degree $leq 1$ remainder in the Euclidean divison of $X^{n}$ by $p_A(X)$ to compute $A^{n}$.



                                For every $ngeq 1$, denote




                                $$
                                X^n=q_n(X)p_A(X)+a_nX+b_n
                                $$




                                the Euclidean division of $X^n$ by $p_A(X)$.
                                Since $X^2equiv 3X+10$ modulo $p_A(X)$, multiplication of the latter by $X$ yields
                                $$
                                cases{a_{n=1}=3a_n+b_n \b_{n+1}=10a_n}iffcases{a_{n+1}=3a_n+10a_{n-1}\b_{n+1}=10a_n}
                                $$
                                Solving for $a_n$ is straightforward given the theory of recurrent linear homogeneous sequences. Given that the roots of the characteristic equation (which is of course $p_A(X)=0$) are $-2$ and $5$, we have $a_n=c (-2)^n+d 5^n$. Considering the initial cases $n=0,1$, we find $c$ and $d$ whence




                                $$a_n=frac{5^n-(-2)^{n}}{7}quadmbox{whence} quad b_n=frac{2cdot 5^{n}+5(-2)^{n}}{7}$$




                                Therefore
                                $$
                                A^n=q_n(A)p_A(A)+a_nA+b_nI_2=a_nA+b_nI_2=pmatrix{a_n+b_n&4a_n\3a_n& 2a_n+b_n}
                                $$
                                hence $A^n$ is equal to




                                $$A^n=pmatrix{frac{3cdot 5^{n}+ (-2)^{n+2}}{7} & frac{4cdot 5^{n}- (-2)^{n+2}}{7} \ frac{3cdot 5^{n}-3cdot (-2)^{n}}{7} & frac{4cdot 5^{n}+3cdot (-2)^{n}}{7}}$$







                                share|cite|improve this answer














                                The well-known strategy is to diagonalize since this matrix is diagonalizable.
                                For a change, here is a slightly different approach which is convenient for small matrices. If you have time, try to perform both methods until the end without computer help. This is fairly equivalent. Although you have two linear systems to solve for eigenvectors, and only one here for the constants $c,d$ below.



                                We will compute $A^n$.



                                Let $p_A(X)=X^2-3X-10$ be the characteristic polynomial of the matrix $A$. By Cayley-Hamilton, $p_A(A)=0$. So it suffices to determine the degree $leq 1$ remainder in the Euclidean divison of $X^{n}$ by $p_A(X)$ to compute $A^{n}$.



                                For every $ngeq 1$, denote




                                $$
                                X^n=q_n(X)p_A(X)+a_nX+b_n
                                $$




                                the Euclidean division of $X^n$ by $p_A(X)$.
                                Since $X^2equiv 3X+10$ modulo $p_A(X)$, multiplication of the latter by $X$ yields
                                $$
                                cases{a_{n=1}=3a_n+b_n \b_{n+1}=10a_n}iffcases{a_{n+1}=3a_n+10a_{n-1}\b_{n+1}=10a_n}
                                $$
                                Solving for $a_n$ is straightforward given the theory of recurrent linear homogeneous sequences. Given that the roots of the characteristic equation (which is of course $p_A(X)=0$) are $-2$ and $5$, we have $a_n=c (-2)^n+d 5^n$. Considering the initial cases $n=0,1$, we find $c$ and $d$ whence




                                $$a_n=frac{5^n-(-2)^{n}}{7}quadmbox{whence} quad b_n=frac{2cdot 5^{n}+5(-2)^{n}}{7}$$




                                Therefore
                                $$
                                A^n=q_n(A)p_A(A)+a_nA+b_nI_2=a_nA+b_nI_2=pmatrix{a_n+b_n&4a_n\3a_n& 2a_n+b_n}
                                $$
                                hence $A^n$ is equal to




                                $$A^n=pmatrix{frac{3cdot 5^{n}+ (-2)^{n+2}}{7} & frac{4cdot 5^{n}- (-2)^{n+2}}{7} \ frac{3cdot 5^{n}-3cdot (-2)^{n}}{7} & frac{4cdot 5^{n}+3cdot (-2)^{n}}{7}}$$








                                share|cite|improve this answer














                                share|cite|improve this answer



                                share|cite|improve this answer








                                edited Dec 8 '13 at 4:08

























                                answered Dec 8 '13 at 3:16









                                Julien

                                38.5k358129




                                38.5k358129























                                    3














                                    Perform an eigenvalue decomposition of $A$, we then get
                                    $$A =
                                    begin{bmatrix}
                                    -4/5 & -1/sqrt2\
                                    3/5 & -1/sqrt2
                                    end{bmatrix}
                                    begin{bmatrix}
                                    -2 & 0\
                                    0 & 5
                                    end{bmatrix}
                                    begin{bmatrix}
                                    -4/5 & -1/sqrt2\
                                    3/5 & -1/sqrt2
                                    end{bmatrix}^{-1}
                                    =VDV^{-1}
                                    $$
                                    where $V = begin{bmatrix}
                                    -4/5 & -1/sqrt2\
                                    3/5 & -1/sqrt2
                                    end{bmatrix}$ and $D = begin{bmatrix}
                                    -2 & 0\
                                    0 & 5
                                    end{bmatrix}$.



                                    Hence,
                                    $$A^n = underbrace{left(VDV^{-1} right)left(VDV^{-1} right)cdots left(VDV^{-1} right)}_{n text{ times}} = VD^n V^{-1}$$






                                    share|cite|improve this answer





















                                    • This is a Jordan decomposition, not an eigenvalue decomposition (nor does an eigenvalue decomposition with a diagonal $D$ exist for non-symmetric/non-Hermitian matrix $A$), because $$V^*V = begin{bmatrix} 1 & (5sqrt{2})^{-1} \ (5sqrt{2})^{-1} & 1 end{bmatrix} ne {rm I}_2.$$
                                      – Vedran Šego
                                      Dec 8 '13 at 3:28
















                                    3














                                    Perform an eigenvalue decomposition of $A$, we then get
                                    $$A =
                                    begin{bmatrix}
                                    -4/5 & -1/sqrt2\
                                    3/5 & -1/sqrt2
                                    end{bmatrix}
                                    begin{bmatrix}
                                    -2 & 0\
                                    0 & 5
                                    end{bmatrix}
                                    begin{bmatrix}
                                    -4/5 & -1/sqrt2\
                                    3/5 & -1/sqrt2
                                    end{bmatrix}^{-1}
                                    =VDV^{-1}
                                    $$
                                    where $V = begin{bmatrix}
                                    -4/5 & -1/sqrt2\
                                    3/5 & -1/sqrt2
                                    end{bmatrix}$ and $D = begin{bmatrix}
                                    -2 & 0\
                                    0 & 5
                                    end{bmatrix}$.



                                    Hence,
                                    $$A^n = underbrace{left(VDV^{-1} right)left(VDV^{-1} right)cdots left(VDV^{-1} right)}_{n text{ times}} = VD^n V^{-1}$$






                                    share|cite|improve this answer





















                                    • This is a Jordan decomposition, not an eigenvalue decomposition (nor does an eigenvalue decomposition with a diagonal $D$ exist for non-symmetric/non-Hermitian matrix $A$), because $$V^*V = begin{bmatrix} 1 & (5sqrt{2})^{-1} \ (5sqrt{2})^{-1} & 1 end{bmatrix} ne {rm I}_2.$$
                                      – Vedran Šego
                                      Dec 8 '13 at 3:28














                                    3












                                    3








                                    3






                                    Perform an eigenvalue decomposition of $A$, we then get
                                    $$A =
                                    begin{bmatrix}
                                    -4/5 & -1/sqrt2\
                                    3/5 & -1/sqrt2
                                    end{bmatrix}
                                    begin{bmatrix}
                                    -2 & 0\
                                    0 & 5
                                    end{bmatrix}
                                    begin{bmatrix}
                                    -4/5 & -1/sqrt2\
                                    3/5 & -1/sqrt2
                                    end{bmatrix}^{-1}
                                    =VDV^{-1}
                                    $$
                                    where $V = begin{bmatrix}
                                    -4/5 & -1/sqrt2\
                                    3/5 & -1/sqrt2
                                    end{bmatrix}$ and $D = begin{bmatrix}
                                    -2 & 0\
                                    0 & 5
                                    end{bmatrix}$.



                                    Hence,
                                    $$A^n = underbrace{left(VDV^{-1} right)left(VDV^{-1} right)cdots left(VDV^{-1} right)}_{n text{ times}} = VD^n V^{-1}$$






                                    share|cite|improve this answer












                                    Perform an eigenvalue decomposition of $A$, we then get
                                    $$A =
                                    begin{bmatrix}
                                    -4/5 & -1/sqrt2\
                                    3/5 & -1/sqrt2
                                    end{bmatrix}
                                    begin{bmatrix}
                                    -2 & 0\
                                    0 & 5
                                    end{bmatrix}
                                    begin{bmatrix}
                                    -4/5 & -1/sqrt2\
                                    3/5 & -1/sqrt2
                                    end{bmatrix}^{-1}
                                    =VDV^{-1}
                                    $$
                                    where $V = begin{bmatrix}
                                    -4/5 & -1/sqrt2\
                                    3/5 & -1/sqrt2
                                    end{bmatrix}$ and $D = begin{bmatrix}
                                    -2 & 0\
                                    0 & 5
                                    end{bmatrix}$.



                                    Hence,
                                    $$A^n = underbrace{left(VDV^{-1} right)left(VDV^{-1} right)cdots left(VDV^{-1} right)}_{n text{ times}} = VD^n V^{-1}$$







                                    share|cite|improve this answer












                                    share|cite|improve this answer



                                    share|cite|improve this answer










                                    answered Dec 8 '13 at 2:35







                                    user17762



















                                    • This is a Jordan decomposition, not an eigenvalue decomposition (nor does an eigenvalue decomposition with a diagonal $D$ exist for non-symmetric/non-Hermitian matrix $A$), because $$V^*V = begin{bmatrix} 1 & (5sqrt{2})^{-1} \ (5sqrt{2})^{-1} & 1 end{bmatrix} ne {rm I}_2.$$
                                      – Vedran Šego
                                      Dec 8 '13 at 3:28


















                                    • This is a Jordan decomposition, not an eigenvalue decomposition (nor does an eigenvalue decomposition with a diagonal $D$ exist for non-symmetric/non-Hermitian matrix $A$), because $$V^*V = begin{bmatrix} 1 & (5sqrt{2})^{-1} \ (5sqrt{2})^{-1} & 1 end{bmatrix} ne {rm I}_2.$$
                                      – Vedran Šego
                                      Dec 8 '13 at 3:28
















                                    This is a Jordan decomposition, not an eigenvalue decomposition (nor does an eigenvalue decomposition with a diagonal $D$ exist for non-symmetric/non-Hermitian matrix $A$), because $$V^*V = begin{bmatrix} 1 & (5sqrt{2})^{-1} \ (5sqrt{2})^{-1} & 1 end{bmatrix} ne {rm I}_2.$$
                                    – Vedran Šego
                                    Dec 8 '13 at 3:28




                                    This is a Jordan decomposition, not an eigenvalue decomposition (nor does an eigenvalue decomposition with a diagonal $D$ exist for non-symmetric/non-Hermitian matrix $A$), because $$V^*V = begin{bmatrix} 1 & (5sqrt{2})^{-1} \ (5sqrt{2})^{-1} & 1 end{bmatrix} ne {rm I}_2.$$
                                    – Vedran Šego
                                    Dec 8 '13 at 3:28











                                    0














                                    $newcommand{+}{^{dagger}}%
                                    newcommand{angles}[1]{leftlangle #1 rightrangle}%
                                    newcommand{braces}[1]{leftlbrace #1 rightrbrace}%
                                    newcommand{bracks}[1]{leftlbrack #1 rightrbrack}%
                                    newcommand{ceil}[1]{,leftlceil #1 rightrceil,}%
                                    newcommand{dd}{{rm d}}%
                                    newcommand{ds}[1]{displaystyle{#1}}%
                                    newcommand{equalby}[1]{{#1 atop {= atop vphantom{huge A}}}}%
                                    newcommand{expo}[1]{,{rm e}^{#1},}%
                                    newcommand{fermi}{,{rm f}}%
                                    newcommand{floor}[1]{,leftlfloor #1 rightrfloor,}%
                                    newcommand{half}{{1 over 2}}%
                                    newcommand{ic}{{rm i}}%
                                    newcommand{iff}{Longleftrightarrow}
                                    newcommand{imp}{Longrightarrow}%
                                    newcommand{isdiv}{,left.rightvert,}%
                                    newcommand{ket}[1]{leftvert #1rightrangle}%
                                    newcommand{ol}[1]{overline{#1}}%
                                    newcommand{pars}[1]{left( #1 right)}%
                                    newcommand{partiald}[3]{frac{partial^{#1} #2}{partial #3^{#1}}}
                                    newcommand{pp}{{cal P}}%
                                    newcommand{root}[2]{,sqrt[#1]{,#2,},}%
                                    newcommand{sech}{,{rm sech}}%
                                    newcommand{sgn}{,{rm sgn}}%
                                    newcommand{totald}[3]{frac{{rm d}^{#1} #2}{{rm d} #3^{#1}}}
                                    newcommand{ul}[1]{underline{#1}}%
                                    newcommand{verts}[1]{leftvert, #1 ,rightvert}$
                                    $$
                                    A = pars{begin{array}{cc}1 & 4 \ 3 & 2end{array}}
                                    =
                                    {3 over 2}
                                    overbrace{pars{begin{array}{cc}1 & 0 \ 0 & 1end{array}}}^{ds{1}}
                                    +
                                    overbrace{{7 over 2}}^{ds{b_{x}}}
                                    overbrace{pars{begin{array}{cc}0 & 1 \ 1 & 0end{array}}}^{ds{sigma_{x}}}
                                    +
                                    overbrace{half,ic}^{ds{b_{y}}}
                                    overbrace{pars{begin{array}{cc}0 & -ic \ ic & 0end{array}}}^{ds{sigma_{y}}}
                                    overbrace{-
                                    half}^{ds{b_{z}}}overbrace{pars{begin{array}{cc}1 & 0 \ 1 & -1end{array}}}^{ds{sigma_{z}}}
                                    =
                                    {3 over 2} + vec{b}cdotvec{sigma}
                                    $$
                                    where $braces{sigma_{ell},, ell = x, y, z}$ are the
                                    Pauli matrices. Then,
                                    $expo{At} = expo{3t/2}expo{vec{b}cdotvec{sigma}t}$. However, $vec{b}cdotvec{sigma}vec{b}cdotvec{sigma} = vec{b}cdotvec{b} = 49/4$ such that
                                    $$
                                    pars{totald[2]{}{t} - {49 over 4}}expo{vec{b}cdotvec{sigma}t} = 0
                                    quadimpquad
                                    expo{vec{b}cdotvec{sigma}t} = muexpo{7t/2} + nuexpo{-7t/2},,quad
                                    mu, nu mbox{are constants},
                                    $$
                                    with $1 = mu + nu$ and $2vec{b}cdotvec{sigma}/7 = mu - nu$ such that
                                    $mu = 1/2 + vec{b}cdotvec{sigma}/7$ and
                                    $nu = 1/2 - vec{b}cdotvec{sigma}/7$:
                                    $$
                                    expo{At} = muexpo{5t} + nuexpo{-2t},,
                                    quadsum_{n = 0}^{infty}{t^{n} over n!},A^{n}
                                    =
                                    sum_{n = 0}^{infty}{t^{n} over n!},
                                    bracks{5^{n}mu + pars{-1}^{n}2^{n}nu}
                                    $$
                                    $$color{#0000ff}{large%
                                    leftlbrace%
                                    begin{array}{rcl}
                                    A^{n} = 5^{n}mu + pars{-1}^{n}2^{n}nu
                                    & = &
                                    halfbracks{5^{n} + pars{-1}^{n}2^{n}}
                                    +
                                    {1 over 7}bracks{5^{n} - pars{-1}^{n}2^{n}}vec{b}cdotvec{sigma}
                                    \[3mm]
                                    vec{b}cdotvec{sigma}
                                    & = &
                                    A - {3 over 2}
                                    =
                                    pars{begin{array}{cc}-1/2 & 5/2 \ 3/2 & 1/2end{array}}
                                    =
                                    halfpars{begin{array}{cc}-1 & 5 \ 3 & 1end{array}}
                                    end{array}right.}
                                    $$






                                    share|cite|improve this answer




























                                      0














                                      $newcommand{+}{^{dagger}}%
                                      newcommand{angles}[1]{leftlangle #1 rightrangle}%
                                      newcommand{braces}[1]{leftlbrace #1 rightrbrace}%
                                      newcommand{bracks}[1]{leftlbrack #1 rightrbrack}%
                                      newcommand{ceil}[1]{,leftlceil #1 rightrceil,}%
                                      newcommand{dd}{{rm d}}%
                                      newcommand{ds}[1]{displaystyle{#1}}%
                                      newcommand{equalby}[1]{{#1 atop {= atop vphantom{huge A}}}}%
                                      newcommand{expo}[1]{,{rm e}^{#1},}%
                                      newcommand{fermi}{,{rm f}}%
                                      newcommand{floor}[1]{,leftlfloor #1 rightrfloor,}%
                                      newcommand{half}{{1 over 2}}%
                                      newcommand{ic}{{rm i}}%
                                      newcommand{iff}{Longleftrightarrow}
                                      newcommand{imp}{Longrightarrow}%
                                      newcommand{isdiv}{,left.rightvert,}%
                                      newcommand{ket}[1]{leftvert #1rightrangle}%
                                      newcommand{ol}[1]{overline{#1}}%
                                      newcommand{pars}[1]{left( #1 right)}%
                                      newcommand{partiald}[3]{frac{partial^{#1} #2}{partial #3^{#1}}}
                                      newcommand{pp}{{cal P}}%
                                      newcommand{root}[2]{,sqrt[#1]{,#2,},}%
                                      newcommand{sech}{,{rm sech}}%
                                      newcommand{sgn}{,{rm sgn}}%
                                      newcommand{totald}[3]{frac{{rm d}^{#1} #2}{{rm d} #3^{#1}}}
                                      newcommand{ul}[1]{underline{#1}}%
                                      newcommand{verts}[1]{leftvert, #1 ,rightvert}$
                                      $$
                                      A = pars{begin{array}{cc}1 & 4 \ 3 & 2end{array}}
                                      =
                                      {3 over 2}
                                      overbrace{pars{begin{array}{cc}1 & 0 \ 0 & 1end{array}}}^{ds{1}}
                                      +
                                      overbrace{{7 over 2}}^{ds{b_{x}}}
                                      overbrace{pars{begin{array}{cc}0 & 1 \ 1 & 0end{array}}}^{ds{sigma_{x}}}
                                      +
                                      overbrace{half,ic}^{ds{b_{y}}}
                                      overbrace{pars{begin{array}{cc}0 & -ic \ ic & 0end{array}}}^{ds{sigma_{y}}}
                                      overbrace{-
                                      half}^{ds{b_{z}}}overbrace{pars{begin{array}{cc}1 & 0 \ 1 & -1end{array}}}^{ds{sigma_{z}}}
                                      =
                                      {3 over 2} + vec{b}cdotvec{sigma}
                                      $$
                                      where $braces{sigma_{ell},, ell = x, y, z}$ are the
                                      Pauli matrices. Then,
                                      $expo{At} = expo{3t/2}expo{vec{b}cdotvec{sigma}t}$. However, $vec{b}cdotvec{sigma}vec{b}cdotvec{sigma} = vec{b}cdotvec{b} = 49/4$ such that
                                      $$
                                      pars{totald[2]{}{t} - {49 over 4}}expo{vec{b}cdotvec{sigma}t} = 0
                                      quadimpquad
                                      expo{vec{b}cdotvec{sigma}t} = muexpo{7t/2} + nuexpo{-7t/2},,quad
                                      mu, nu mbox{are constants},
                                      $$
                                      with $1 = mu + nu$ and $2vec{b}cdotvec{sigma}/7 = mu - nu$ such that
                                      $mu = 1/2 + vec{b}cdotvec{sigma}/7$ and
                                      $nu = 1/2 - vec{b}cdotvec{sigma}/7$:
                                      $$
                                      expo{At} = muexpo{5t} + nuexpo{-2t},,
                                      quadsum_{n = 0}^{infty}{t^{n} over n!},A^{n}
                                      =
                                      sum_{n = 0}^{infty}{t^{n} over n!},
                                      bracks{5^{n}mu + pars{-1}^{n}2^{n}nu}
                                      $$
                                      $$color{#0000ff}{large%
                                      leftlbrace%
                                      begin{array}{rcl}
                                      A^{n} = 5^{n}mu + pars{-1}^{n}2^{n}nu
                                      & = &
                                      halfbracks{5^{n} + pars{-1}^{n}2^{n}}
                                      +
                                      {1 over 7}bracks{5^{n} - pars{-1}^{n}2^{n}}vec{b}cdotvec{sigma}
                                      \[3mm]
                                      vec{b}cdotvec{sigma}
                                      & = &
                                      A - {3 over 2}
                                      =
                                      pars{begin{array}{cc}-1/2 & 5/2 \ 3/2 & 1/2end{array}}
                                      =
                                      halfpars{begin{array}{cc}-1 & 5 \ 3 & 1end{array}}
                                      end{array}right.}
                                      $$






                                      share|cite|improve this answer


























                                        0












                                        0








                                        0






                                        $newcommand{+}{^{dagger}}%
                                        newcommand{angles}[1]{leftlangle #1 rightrangle}%
                                        newcommand{braces}[1]{leftlbrace #1 rightrbrace}%
                                        newcommand{bracks}[1]{leftlbrack #1 rightrbrack}%
                                        newcommand{ceil}[1]{,leftlceil #1 rightrceil,}%
                                        newcommand{dd}{{rm d}}%
                                        newcommand{ds}[1]{displaystyle{#1}}%
                                        newcommand{equalby}[1]{{#1 atop {= atop vphantom{huge A}}}}%
                                        newcommand{expo}[1]{,{rm e}^{#1},}%
                                        newcommand{fermi}{,{rm f}}%
                                        newcommand{floor}[1]{,leftlfloor #1 rightrfloor,}%
                                        newcommand{half}{{1 over 2}}%
                                        newcommand{ic}{{rm i}}%
                                        newcommand{iff}{Longleftrightarrow}
                                        newcommand{imp}{Longrightarrow}%
                                        newcommand{isdiv}{,left.rightvert,}%
                                        newcommand{ket}[1]{leftvert #1rightrangle}%
                                        newcommand{ol}[1]{overline{#1}}%
                                        newcommand{pars}[1]{left( #1 right)}%
                                        newcommand{partiald}[3]{frac{partial^{#1} #2}{partial #3^{#1}}}
                                        newcommand{pp}{{cal P}}%
                                        newcommand{root}[2]{,sqrt[#1]{,#2,},}%
                                        newcommand{sech}{,{rm sech}}%
                                        newcommand{sgn}{,{rm sgn}}%
                                        newcommand{totald}[3]{frac{{rm d}^{#1} #2}{{rm d} #3^{#1}}}
                                        newcommand{ul}[1]{underline{#1}}%
                                        newcommand{verts}[1]{leftvert, #1 ,rightvert}$
                                        $$
                                        A = pars{begin{array}{cc}1 & 4 \ 3 & 2end{array}}
                                        =
                                        {3 over 2}
                                        overbrace{pars{begin{array}{cc}1 & 0 \ 0 & 1end{array}}}^{ds{1}}
                                        +
                                        overbrace{{7 over 2}}^{ds{b_{x}}}
                                        overbrace{pars{begin{array}{cc}0 & 1 \ 1 & 0end{array}}}^{ds{sigma_{x}}}
                                        +
                                        overbrace{half,ic}^{ds{b_{y}}}
                                        overbrace{pars{begin{array}{cc}0 & -ic \ ic & 0end{array}}}^{ds{sigma_{y}}}
                                        overbrace{-
                                        half}^{ds{b_{z}}}overbrace{pars{begin{array}{cc}1 & 0 \ 1 & -1end{array}}}^{ds{sigma_{z}}}
                                        =
                                        {3 over 2} + vec{b}cdotvec{sigma}
                                        $$
                                        where $braces{sigma_{ell},, ell = x, y, z}$ are the
                                        Pauli matrices. Then,
                                        $expo{At} = expo{3t/2}expo{vec{b}cdotvec{sigma}t}$. However, $vec{b}cdotvec{sigma}vec{b}cdotvec{sigma} = vec{b}cdotvec{b} = 49/4$ such that
                                        $$
                                        pars{totald[2]{}{t} - {49 over 4}}expo{vec{b}cdotvec{sigma}t} = 0
                                        quadimpquad
                                        expo{vec{b}cdotvec{sigma}t} = muexpo{7t/2} + nuexpo{-7t/2},,quad
                                        mu, nu mbox{are constants},
                                        $$
                                        with $1 = mu + nu$ and $2vec{b}cdotvec{sigma}/7 = mu - nu$ such that
                                        $mu = 1/2 + vec{b}cdotvec{sigma}/7$ and
                                        $nu = 1/2 - vec{b}cdotvec{sigma}/7$:
                                        $$
                                        expo{At} = muexpo{5t} + nuexpo{-2t},,
                                        quadsum_{n = 0}^{infty}{t^{n} over n!},A^{n}
                                        =
                                        sum_{n = 0}^{infty}{t^{n} over n!},
                                        bracks{5^{n}mu + pars{-1}^{n}2^{n}nu}
                                        $$
                                        $$color{#0000ff}{large%
                                        leftlbrace%
                                        begin{array}{rcl}
                                        A^{n} = 5^{n}mu + pars{-1}^{n}2^{n}nu
                                        & = &
                                        halfbracks{5^{n} + pars{-1}^{n}2^{n}}
                                        +
                                        {1 over 7}bracks{5^{n} - pars{-1}^{n}2^{n}}vec{b}cdotvec{sigma}
                                        \[3mm]
                                        vec{b}cdotvec{sigma}
                                        & = &
                                        A - {3 over 2}
                                        =
                                        pars{begin{array}{cc}-1/2 & 5/2 \ 3/2 & 1/2end{array}}
                                        =
                                        halfpars{begin{array}{cc}-1 & 5 \ 3 & 1end{array}}
                                        end{array}right.}
                                        $$






                                        share|cite|improve this answer














                                        $newcommand{+}{^{dagger}}%
                                        newcommand{angles}[1]{leftlangle #1 rightrangle}%
                                        newcommand{braces}[1]{leftlbrace #1 rightrbrace}%
                                        newcommand{bracks}[1]{leftlbrack #1 rightrbrack}%
                                        newcommand{ceil}[1]{,leftlceil #1 rightrceil,}%
                                        newcommand{dd}{{rm d}}%
                                        newcommand{ds}[1]{displaystyle{#1}}%
                                        newcommand{equalby}[1]{{#1 atop {= atop vphantom{huge A}}}}%
                                        newcommand{expo}[1]{,{rm e}^{#1},}%
                                        newcommand{fermi}{,{rm f}}%
                                        newcommand{floor}[1]{,leftlfloor #1 rightrfloor,}%
                                        newcommand{half}{{1 over 2}}%
                                        newcommand{ic}{{rm i}}%
                                        newcommand{iff}{Longleftrightarrow}
                                        newcommand{imp}{Longrightarrow}%
                                        newcommand{isdiv}{,left.rightvert,}%
                                        newcommand{ket}[1]{leftvert #1rightrangle}%
                                        newcommand{ol}[1]{overline{#1}}%
                                        newcommand{pars}[1]{left( #1 right)}%
                                        newcommand{partiald}[3]{frac{partial^{#1} #2}{partial #3^{#1}}}
                                        newcommand{pp}{{cal P}}%
                                        newcommand{root}[2]{,sqrt[#1]{,#2,},}%
                                        newcommand{sech}{,{rm sech}}%
                                        newcommand{sgn}{,{rm sgn}}%
                                        newcommand{totald}[3]{frac{{rm d}^{#1} #2}{{rm d} #3^{#1}}}
                                        newcommand{ul}[1]{underline{#1}}%
                                        newcommand{verts}[1]{leftvert, #1 ,rightvert}$
                                        $$
                                        A = pars{begin{array}{cc}1 & 4 \ 3 & 2end{array}}
                                        =
                                        {3 over 2}
                                        overbrace{pars{begin{array}{cc}1 & 0 \ 0 & 1end{array}}}^{ds{1}}
                                        +
                                        overbrace{{7 over 2}}^{ds{b_{x}}}
                                        overbrace{pars{begin{array}{cc}0 & 1 \ 1 & 0end{array}}}^{ds{sigma_{x}}}
                                        +
                                        overbrace{half,ic}^{ds{b_{y}}}
                                        overbrace{pars{begin{array}{cc}0 & -ic \ ic & 0end{array}}}^{ds{sigma_{y}}}
                                        overbrace{-
                                        half}^{ds{b_{z}}}overbrace{pars{begin{array}{cc}1 & 0 \ 1 & -1end{array}}}^{ds{sigma_{z}}}
                                        =
                                        {3 over 2} + vec{b}cdotvec{sigma}
                                        $$
                                        where $braces{sigma_{ell},, ell = x, y, z}$ are the
                                        Pauli matrices. Then,
                                        $expo{At} = expo{3t/2}expo{vec{b}cdotvec{sigma}t}$. However, $vec{b}cdotvec{sigma}vec{b}cdotvec{sigma} = vec{b}cdotvec{b} = 49/4$ such that
                                        $$
                                        pars{totald[2]{}{t} - {49 over 4}}expo{vec{b}cdotvec{sigma}t} = 0
                                        quadimpquad
                                        expo{vec{b}cdotvec{sigma}t} = muexpo{7t/2} + nuexpo{-7t/2},,quad
                                        mu, nu mbox{are constants},
                                        $$
                                        with $1 = mu + nu$ and $2vec{b}cdotvec{sigma}/7 = mu - nu$ such that
                                        $mu = 1/2 + vec{b}cdotvec{sigma}/7$ and
                                        $nu = 1/2 - vec{b}cdotvec{sigma}/7$:
                                        $$
                                        expo{At} = muexpo{5t} + nuexpo{-2t},,
                                        quadsum_{n = 0}^{infty}{t^{n} over n!},A^{n}
                                        =
                                        sum_{n = 0}^{infty}{t^{n} over n!},
                                        bracks{5^{n}mu + pars{-1}^{n}2^{n}nu}
                                        $$
                                        $$color{#0000ff}{large%
                                        leftlbrace%
                                        begin{array}{rcl}
                                        A^{n} = 5^{n}mu + pars{-1}^{n}2^{n}nu
                                        & = &
                                        halfbracks{5^{n} + pars{-1}^{n}2^{n}}
                                        +
                                        {1 over 7}bracks{5^{n} - pars{-1}^{n}2^{n}}vec{b}cdotvec{sigma}
                                        \[3mm]
                                        vec{b}cdotvec{sigma}
                                        & = &
                                        A - {3 over 2}
                                        =
                                        pars{begin{array}{cc}-1/2 & 5/2 \ 3/2 & 1/2end{array}}
                                        =
                                        halfpars{begin{array}{cc}-1 & 5 \ 3 & 1end{array}}
                                        end{array}right.}
                                        $$







                                        share|cite|improve this answer














                                        share|cite|improve this answer



                                        share|cite|improve this answer








                                        edited Dec 8 '13 at 5:26

























                                        answered Dec 8 '13 at 4:04









                                        Felix Marin

                                        67.2k7107141




                                        67.2k7107141























                                            0














                                            Two additional methods that you can use once you know the eigenvalues $lambda_1$ and $lambda_2$:




                                            • Decompose $A$ into $lambda_1P_1+lambda_2P_2$, where $P_1 = {A-lambda_2Ioverlambda_2-lambda_1}$ and $P_2={A-lambda_1Ioverlambda_1-lambda_2}$ are projections onto the corresponding eigenspaces with $P_1P_2=P_2P_1=0$. If you expand $A^{1000}$ using the binomial theorem, you’ll find that all but two terms vanish, giving $lambda_1^{1000}P_1+lambda_2^{1000}P_2$.

                                            • Use the Cayley-Hamilton theorem to write $A^{1000}=aI+bA$ for some undetermined coefficients $a$ and $b$, then use the fact that this equation is also satisfied by the eigenvalues, which gives you the system of linear equations $a+blambda_i=lambda_i^{100}$ to solve for $a$ and $b$.


                                            When $A$ has repeated eigenvalues, you’ll need to modify the above methods a bit, but the underlying ideas are still the same.






                                            share|cite|improve this answer


























                                              0














                                              Two additional methods that you can use once you know the eigenvalues $lambda_1$ and $lambda_2$:




                                              • Decompose $A$ into $lambda_1P_1+lambda_2P_2$, where $P_1 = {A-lambda_2Ioverlambda_2-lambda_1}$ and $P_2={A-lambda_1Ioverlambda_1-lambda_2}$ are projections onto the corresponding eigenspaces with $P_1P_2=P_2P_1=0$. If you expand $A^{1000}$ using the binomial theorem, you’ll find that all but two terms vanish, giving $lambda_1^{1000}P_1+lambda_2^{1000}P_2$.

                                              • Use the Cayley-Hamilton theorem to write $A^{1000}=aI+bA$ for some undetermined coefficients $a$ and $b$, then use the fact that this equation is also satisfied by the eigenvalues, which gives you the system of linear equations $a+blambda_i=lambda_i^{100}$ to solve for $a$ and $b$.


                                              When $A$ has repeated eigenvalues, you’ll need to modify the above methods a bit, but the underlying ideas are still the same.






                                              share|cite|improve this answer
























                                                0












                                                0








                                                0






                                                Two additional methods that you can use once you know the eigenvalues $lambda_1$ and $lambda_2$:




                                                • Decompose $A$ into $lambda_1P_1+lambda_2P_2$, where $P_1 = {A-lambda_2Ioverlambda_2-lambda_1}$ and $P_2={A-lambda_1Ioverlambda_1-lambda_2}$ are projections onto the corresponding eigenspaces with $P_1P_2=P_2P_1=0$. If you expand $A^{1000}$ using the binomial theorem, you’ll find that all but two terms vanish, giving $lambda_1^{1000}P_1+lambda_2^{1000}P_2$.

                                                • Use the Cayley-Hamilton theorem to write $A^{1000}=aI+bA$ for some undetermined coefficients $a$ and $b$, then use the fact that this equation is also satisfied by the eigenvalues, which gives you the system of linear equations $a+blambda_i=lambda_i^{100}$ to solve for $a$ and $b$.


                                                When $A$ has repeated eigenvalues, you’ll need to modify the above methods a bit, but the underlying ideas are still the same.






                                                share|cite|improve this answer












                                                Two additional methods that you can use once you know the eigenvalues $lambda_1$ and $lambda_2$:




                                                • Decompose $A$ into $lambda_1P_1+lambda_2P_2$, where $P_1 = {A-lambda_2Ioverlambda_2-lambda_1}$ and $P_2={A-lambda_1Ioverlambda_1-lambda_2}$ are projections onto the corresponding eigenspaces with $P_1P_2=P_2P_1=0$. If you expand $A^{1000}$ using the binomial theorem, you’ll find that all but two terms vanish, giving $lambda_1^{1000}P_1+lambda_2^{1000}P_2$.

                                                • Use the Cayley-Hamilton theorem to write $A^{1000}=aI+bA$ for some undetermined coefficients $a$ and $b$, then use the fact that this equation is also satisfied by the eigenvalues, which gives you the system of linear equations $a+blambda_i=lambda_i^{100}$ to solve for $a$ and $b$.


                                                When $A$ has repeated eigenvalues, you’ll need to modify the above methods a bit, but the underlying ideas are still the same.







                                                share|cite|improve this answer












                                                share|cite|improve this answer



                                                share|cite|improve this answer










                                                answered 2 days ago









                                                amd

                                                29.3k21050




                                                29.3k21050






























                                                    draft saved

                                                    draft discarded




















































                                                    Thanks for contributing an answer to Mathematics Stack Exchange!


                                                    • Please be sure to answer the question. Provide details and share your research!

                                                    But avoid



                                                    • Asking for help, clarification, or responding to other answers.

                                                    • Making statements based on opinion; back them up with references or personal experience.


                                                    Use MathJax to format equations. MathJax reference.


                                                    To learn more, see our tips on writing great answers.





                                                    Some of your past answers have not been well-received, and you're in danger of being blocked from answering.


                                                    Please pay close attention to the following guidance:


                                                    • Please be sure to answer the question. Provide details and share your research!

                                                    But avoid



                                                    • Asking for help, clarification, or responding to other answers.

                                                    • Making statements based on opinion; back them up with references or personal experience.


                                                    To learn more, see our tips on writing great answers.




                                                    draft saved


                                                    draft discarded














                                                    StackExchange.ready(
                                                    function () {
                                                    StackExchange.openid.initPostLogin('.new-post-login', 'https%3a%2f%2fmath.stackexchange.com%2fquestions%2f597602%2ffinding-a-2x2-matrix-raised-to-the-power-of-1000%23new-answer', 'question_page');
                                                    }
                                                    );

                                                    Post as a guest















                                                    Required, but never shown





















































                                                    Required, but never shown














                                                    Required, but never shown












                                                    Required, but never shown







                                                    Required, but never shown

































                                                    Required, but never shown














                                                    Required, but never shown












                                                    Required, but never shown







                                                    Required, but never shown







                                                    Popular posts from this blog

                                                    An IMO inspired problem

                                                    Management

                                                    Has there ever been an instance of an active nuclear power plant within or near a war zone?